• Không có kết quả nào được tìm thấy

Một số bài toán hay ôn thi VMO năm 2023 có lời giải chi tiết

N/A
N/A
Protected

Academic year: 2022

Chia sẻ "Một số bài toán hay ôn thi VMO năm 2023 có lời giải chi tiết"

Copied!
63
0
0

Loading.... (view fulltext now)

Văn bản

(1)

CHUYÊN TO ÁN - CHUYÊN BẾN TRE

THỈNH THOẢNG,

MỘT BÀI HAY 2022

TỔNG HỢP CÁC BÀI TOÁN HAY

CHUYÊN TOÁN - CHUYÊN BẾN TRE

Vol. 1

(2)

CHUYÊN TO ÁN - CHUYÊN BẾN TRE

Biên tập: Nguyễn Song Thiên Long (S

KY

D

RA

)

Ngày 16 tháng 3 năm 2022

Tóm tắt nội dung

Trong tài liệu này, tôi sẽ tổng hợp các bài toán hay được đề xuất bởi các học sinh cũng như cựu học sinh trường THPT Chuyên Bến Tre. Các bài toán xoay quanh các chủ đề thường gặp trong các kì thi Olympic Toán cũng như HSG Toán, có mức độ dễ khó khác nhau. Hi vọng tài liệu này sẽ giúp ích một phần nhỏ cho bạn đọc trong việc học và giải Toán.

Các bài toán sẽ được phân loại theo 4 chủ đề chính: Đại số(Algebra), Số học(Number theory), Hình học(Geometry)và Tổ hợp(Combinatorics).

Mục đích của tài liệu này là nhằm chia sẻ kiến thức đến cho các bạn học sinh, cũng như tạo cơ hội để liên kết các thành viên trong khối Toán. Xin gửi lời cám ơn chân thành đến các anh chị và các bạn đã tham gia đóng góp trong lần này và hi vọng rằng Volume 2sẽ có thể kết nối được với nhiều thành viên hơn nữa!

Và bây giờ, mời bạn đọc đến vớiThỉnh thoảng, một bài hay - Tổng hợp các bài toán hay, Số 1!

1 Đại số

Bài toán A1 (HSG Bình Dương 2022) Giải phương trình sau trên tập số thực (x−4)√

x−2−1

√4−x+x5 = 2+ (2x−4)√ x−2 x−1

Đề xuất bởi Phạm Khánh Duy, 11T

Lời giải. Điều kiện xác định:

( √ 2⩽x4

4−x+x5̸= 0 ⇔ 2⩽ x4 (Do √

4−x+x−5 = − √

4−x2

+√

4−x−1 = − √

4−x− 1 2

2

3

4 < 0 với mọix ⩽4)

(3)

TỔNG HỢP CÁ C BÀI TO ÁN HA Y

Phương trình tương đương (x−1)h(x−4)√

x−21i = h2+ (2x−4)√

x−2i

4−x+x−5

x25x+4√

x−2x+1= 2√

4−x+2x−10+ (2x−4)√

x−2√ 4−x +2x2−14x+20

x−2

x2−9x+16

x−2+ (2x−4)√

x−2

4−x+2

4−x+3x−11 = 0 Áp dụng bất đẳng thức AM-GM:

















√x−2= 1.√

x−2⩽ 1

2+ √

x−22

2 = x−1

2

√x−2√

4−x⩽

√x−22

+ √

4−x2

2 = 1

√4−x= 1.√

4−x⩽ 1

2+ √

4−x2

2 = 5−x

2

Do đóVT ⩽ x2−9x+16

· x1

2 + (2x−4).1+2·5x

2 +3x−11

= x

3−10x2+33x−36

2 = (x−3)2(x−4)

2 ⩽ 0với mọix ∈ [2; 4]. Dấu “=” xảy ra khi và chỉ khi

( √(x−3)2(x−4) = 0 x−2= √

4−x =1 ⇔ x =3.

Thử lại thấyx = 3là nghiệm.

Vậy phương trình có nghiệm duy nhất x= 3.

Bài toán A2

Giải hệ phương trình nghiệm nguyên sau





2022x =2022y2022+2022log2022(2022y) 2022y = 2022z−2022+2022log2022(2022z) 2022z =2022x−2022+2022log2022(2022x)

Đề xuất bởi Nguyễn Song Thiên Long, 11T Lời giải. Điều kiện xác định:x,y,z>0.

Xét hàm số f(t) = 2022t g(t) = 2022t−2022+2022 log2022(2022t), t > 0thì hai hàm số này đồng biến trên(0;+).

(4)

CHUYÊN TO ÁN - CHUYÊN BẾN TRE

Hệ phương trình đã cho viết lại dưới dạng





f(x) = g(y) f(y) = g(z) f(z) = g(x)

Không mất tính tổng quát, giả sử x= max{x,y,z}. Khi đó x⩾ y, x⩾ z.

Do

x ⩾y ⇒ f(x)⩾ f(y) ⇒ g(y) ⩾ g(z) ⇒ y ⩾z

⇒ f(y) ⩾ f(z) ⇒ g(z)⩾ g(x) ⇒ z⩾ x ⇒ x = y= z.

Đưa về giải phương trình:

2022t =2022t−2022+2022 log2022(2022t) (*) Đặth(t) = 2022t−2022t−2022 log2022t.

Tính được h(t) = 0tại duy nhất một điểm t0, h(t) đổi dấu từ âm sang dương qua điểmt0, vàh(t0) <0. Như vậy phương trìnhh(t) = 0có đúng 2 nghiệm.

Dễ thấyh(1) = 0.

Chứng minh được h(t) liên tục trên đoạn 1

2022;1 2

và h 1

2022

.h 1

2

< 0. Khi đó, tồn tại ít nhất một điểmλ

1 2022;1

2

sao choh(λ) = 0.

Vậy ta suy ra được t = 1 là nghiệm nguyên duy nhất của phương trình (*), hay (x,y,z) = (1, 1, 1)là nghiệm nguyên duy nhất của hệ phương trình. □

Bài toán A3

Giải hệ phương trình sau:





x2−yz = |y−z|+1 y2−zx = |z−x|+1 z2−xy = |x−y|+1

Đề xuất bởi Đoàn Quang Đăng, 12T Lời giải.Không mất tính tổng quát, giả sử x⩾ y⩾ z.

Khi đó, hệ phương trình trở thành





x2yz =y−z+1 (1) y2−zx =x−z+1 (2) z2−xy =x−y+1 (3)

(5)

TỔNG HỢP CÁ C BÀI TO ÁN HA Y

Trừ vế theo vế của phương trình (1) cho phương trình (2), ta được x2−y2+z(x−y) = y−x, hay(x−y)(x+y+z+1) = 0 Suy rax =y∨ x+y+z= −1.

Trừ vế theo vế của phương trình (2) cho phương trình (3), ta được y2−z2+x(y−z) = y−z, hay (y−z)(x+y+z−1) = 0 Suy ray= z∨x+y+z =1.

• Trường hợpx = y: Nếu x= y =zthì phương trình (1) trở thành0 =1(vô lí).

Vậy y ̸= z, suy ra x+y+z = 1, hay 2x+z = 1. Thay y = x và z = 1−2x vào phương trình (1), ta được

x2x(1−2x) = z−(1−2x) +1⇔ 3x24x= 0⇔ x= 0∨x= 4 3 Với x =0, tìm đượcy =0, z= 1(loại do x ⩾yz).

Với x = 4

3, tìm được y= 4

3, z =−5

3 (thỏa).

• Trường hợpx ̸=y: Ta cóx+y+z = −1nên suy ra đượcy = z. Khi đó,x+2z =−1, hayx = −1−2z, phương trình (1) trở thành

(−1−2z)2 −z2 = 1⇔ 3z2+4z =0 ⇔ z= 0∨z =−4 3 Với z=0, tìm đượcy =0, x= −1(loại doxyz).

Với z= −4

3, tìm đượcy =−4

3, x = 5

3 (thỏa).

Do hệ phương trình đối xứng nên xét các hoán vị của 2 nghiệm trên, ta có (x,y,z) ∈

4 3,4

3,−5 3

; 4

3,−5 3,4

3

;

5 3,4

3,4 3

; 5

3,−4 3,−4

3

;

4 3,5

3,−4 3

;

4 3,−4

3,5 3

(6)

CHUYÊN TO ÁN - CHUYÊN BẾN TRE

Bài toán A4 (British MO 2022)

Tìm tất cả hàm số f :Z+Z+ thỏa mãn

2b f(f(a2) +a) = f(a+1)f(2ab) với mọi số nguyên dươnga, b.

Đề xuất bởi Đoàn Quang Đăng, 12T Lời giải. Giả sử tồn tại hàm số thỏa mãn yêu cầu bài toán.

Ký hiệu P(a,b)chỉ khẳng định2b f(f(a2) +a) = f(a+1)f(2ab) ∀a,b ∈ Z+. Từ P(1,b)ta suy ra

2b f(f(1) +1) = f(2)f(2b) ⇒ f(2b) = 2bc ∀b ∈ Z+. vớic = f(f(1) +1)

f(2) là hằng số.

Với klà số nguyên dương, từ P(2k, 1)ta suy ra

2f(f(4k2) +2k) = f(2k+1)f(2k) ⇒ f(2k+1) = 2kc+1.

Ta chọnalà số nguyên dương chẵn,blà số nguyên dương lẻ, khi đó từ P(a,b)ta suy ra (a−1)c+1 = ac hay c = 1. Thay vào ta suy ra f(x) = x với mọi số nguyên dương x >1.

Chú ý rằng f(f(1) +1) = f(2) nên f(1) +1 = 2hay f(1) = 1. Như vậy, ta kết luận được f(x) = x với mọi số nguyên dương x. Thử lại ta thấy hàm số này thỏa mãn yêu

cầu bài toán. □

Bài toán A5 (Ireland 2021)

Cho hàm số g: [0;+) → [0;+)thỏa mãn g(g(x)) = 3x

x+3 ∀x >0.

Giả sửg(x) = x+1

2 ∀x ∈ [2; 3].Tính g(2022)g 1

2022

.

Đề xuất bởi Đoàn Quang Đăng, 12T Trước hết, ta sẽ xác địnhg(x)với mọix⩾6.

Nhận xét. g(x) = 12x+9

2x+15 ∀x⩾6.

Chứng minh.Chú ý rằng 2< 3x

x+3 <3 ∀x⩾6nên

(7)

TỔNG HỢP CÁ C BÀI TO ÁN HA Y

g(g(g(x))) = g

3x x+3

= 1 2

3x x+3 +1

= 4x+3

2x+6 ∀x6.

Mặt khác, thay xbởig(x)vào phương trình ban đầu ta được g(g(g(x))) = 3g(x)

g(x) +3 ∀x⩾6.

So sánh hai đẳng thức trên ta suy ra 1

3 + 1

g(x) = 2x+6

4x+3 ∀x >6 hay

g(x) = 12x+9

2x+15 ∀x⩾6.

Từ đây thay x= 2021ta đượcg(2021) = 12.2022+9 2.2022+15 Nhận xét.g

6 n

= 6

n+1 ∀n∈ N,n> 2.

Chứng minh.Ta dễ dàng tính được g(3) = 2,g(2) = 3

2.Khi đó g

3 2

= g(g(2)) = 3.2

2+3 = 6 5

g 6

5

= g

g 3

2

= 3.3

2 3 2+3

=1

g(1) = g

g 6

5

= 3.6

5 6 5+3

= 6 7.

Ta chứng minh mệnh đề bằng phương pháp quy nạp. Với n = 2, 3mệnh đề đúng.

Giả sử mệnh đề đúng đếnn−1.Khi đó

g 6

n

= g

g 6

n

=

3. 6 n−1 6

n−1+3

= 18

6+3n−3 = 6 n+10. Chứng minh hoàn tất. Từ đây ta thayn =6.2022thì tính được

g 1

2022

= 6 12142.

(8)

CHUYÊN TO ÁN - CHUYÊN BẾN TRE

Bài toán A6 (Thais TST Training)

Tìm tất cả hàm số f :RRthỏa mãn

f (max{x,y}+max{f(x),f(y)}) = x+y với mọix,y ∈ R.

Đề xuất bởi Đoàn Quang Đăng, 12T Lời giải. Giả sử tồn tại hàm số thỏa mãn yêu cầu bài toán.

Ký hiệu P(x,y)chỉ khẳng định f (max{x,y}+max{f(x), f(y)}) = x+y, ∀x,y ∈ R. Đặta = f(0).Từ P(x,x)ta suy ra

f(x+ f(x)) =2x, ∀x ∈ R.

Giả sửx,ylà hai số thực thỏa mãnx > y.Nếu f(x) ⩽ f(y)thì từP(x,y)ta suy ra f(x+ f(x)) = x+y=2x ⇒ x =y,

điều này vô lý do x > y. Như vậy với x > y kéo theo f(x) > f(y). Ta viết P(x,y) lại thành

f(x+ f(y)) = x+y, ∀x> y. (1) Từ (1) ta thayy =0thì được f(x+a) = x, ∀x >0,suy ra

f(x) = x−a, ∀x >a. (2) Với số thực y, ta chọn x đủ lớn sao cho x > max{y,a− f(y)}. Khi đó với x > y thì từ (1) ta suy ra f(x+ f(y)) = x+y và với x+ f(y) > a, từ (2) suy ra f(x+ f(y)) = x+ f(y)−a.So sánh hai đẳng thức này ta thu được

f(y) = y+a, ∀y ∈ R.

Thay vào phương trình ban đầu ta dễ dàng tìm được a=0.

Vậy tất cả hàm số thỏa mãn yêu cầu bài toán là f(x) = x, ∀x ∈ R.

(9)

TỔNG HỢP CÁ C BÀI TO ÁN HA Y

Bài toán A7 (USAMO 2019)

Cho hàm số f :Z+Z+ thỏa mãn

f f(n)(n) = n

2

f2(n) (1)

với mọi số nguyên dươngn,trong đó fk(n) = f(f(. . .(n). . .)) k lần tác động f vàon.Tìm tất cả giá trị có thể của f(2022).

Đề xuất bởi Đoàn Quang Đăng, 12T Lời giải. Trước hết dễ thấy f là đơn ánh. Thayn= 1vào phương trình (1) ta được

f f(1)f2(1) = 1 ⇒ f2(1) = 1.

Đặt f(1) = avà thayn = avào phương trình (1), khi đó ff(a)(a)f2(a) = a2 ⇒ f(a) = a.

Như vậy f(a) = f(1),kết hợp với tính đơn ánh ta suy raa= 1hay f(1) = 1.

Tiếp theo, ta chứng minh bằng quy nạp rằng

f(n) = nvới mọinnguyên dương lẻ. (2) Rõ ràng (2) đúng vớin= 1.Giả sử (2) đúngn−2,trong đón3.Ta cần chứng minh (2) đúng vớin.Thật vậy, theo giả thiết ta có

ff(n)(n)f2(n) = n2.

Từ đây ta nhận xét rằng cả ff(n)(n) f2(n)đều là số lẻ. Hơn nữa, theo giả thiết quy nạp và f là đơn ánh nên ff(n)(n) ⩾nvà f2(n)⩾ n.Do đó

ff(n)(n) = f2(n) = n.

Đến đây ta xét hai trường hợp

• Nếu f(n)là số lẻ thì sử dụng tính đơn ánh của f ta được ff(n)(n) = n= ff(n)+1(n) ⇒ f(n) = n.

• Nếu f(n) = bchẵn, thì f(b) = f(f(n)) =n =2k+1(k⩾1)và từ f2(n) = nta suy rablà điểm bất động của hàm số f2.Thayn =bvào phương trình (1) thì được

f f(b)(b)f2(b) = b2 ⇒ f2k+1(b).b = f

f2k(b).b = f(b).b = b2 ⇒ f(b) = b = f(n). Do tính đơn ánh nên ta suy ran =b là số chẵn, vô lý.

(10)

CHUYÊN TO ÁN - CHUYÊN BẾN TRE

Do đó (2) đúng với mọinlẻ. Tóm lại f(n) = nvới mọi số tự nhiên lẻn.Từ đây suy ra f(2022)có thể nhận các giá trị chẵn trênZ+.Một ví dụ để chứng tỏ rằng f(2022)có thể nhận bất kì giá trịλchẵn nào, đó là

f(n) =





n nếu n ̸=2022,λ λ nếu n =2022 2022 nếu n =λ

□ Bài toán A8 (Russia 2008)

Cho hai dãy(an)(bn)thỏa mãn





an+1 = 1+an+anbn

bn ; a1 =1 bn+1 = 1+bn+anbn

an ; b1 = 2 Chứng minh rằng(an)hội tụ.

Đề xuất bởi Đào Trọng Toàn, 12T Lời giải.Dễ thấy(an),(bn)là các dãy số dương.

Ta có biến đổi sau:

1

bn+1+1 − 1

an+1+1 = an

(1+an)(1+bn) − bn

(1+an)(1+bn) = 1

bn+1 − 1 an+1

1

bn+11

an +1 = const= 1

2+11

1+1 =−1 6

1

an+1 = 1

bn+1+ 1 6 > 1

6

⇒ an <5, ∀n ∈ N

Chú ý rằng an+1 = an +1+an

bn > an.

Khi đó,(an)tăng và bị chặn trên bởi 5 nên hội tụ. □

(11)

TỔNG HỢP CÁ C BÀI TO ÁN HA Y

Bài toán A9 (Chọn đội tuyển VMO Lâm Đồng 2022) Đặt f(n) = n2+n+12

+1. Cho an = f(1).f(3). . . f(2n−1)

f(2).f(4). . . f(2n) với nlà số nguyên dương. Tìmlimn√

an.

Đề xuất bởi Trần Đinh Vĩnh Thụy, T15-18

Lời giải. Ta có

f(n) = n2+n+12

+1 =n4+2n3+3n2+2n+2= (n2+1)(n2+2n+2). Xét tỉ số

f(n)

f(n+1) = (n2+1)(n2+2n+2) h(n+1)2+1i h

(n+1)2+2(n+1) +2i = (n2+1)(n2+2n+2) (n2+2n+2)(n2+4n+5)

= n

2 +1

(n+2)2+1 (*) Áp dụng (*) chonsố lẻ1, 3, . . . , 2n−1, ta được

f(1) f(2) = 1

2+1

32+1, f(3) f(4) = 3

2 +1

52 +1, . . . , f(2n−1)

f(2n) = (2n−1)2 +1 (2n+1)2 +1 Nhân vế theo vế của các đẳng thức trên, ta có

f(1) f(2).

f(3) f(4). . .

f(2n−1) f(2n) = 1

2+1 32+1.

32+1 52+1. . .

(2n−1)2+1 (2n+1)2+1 hay

an = 2 4n2 +4n+2 Vậylimn√

an =limn.

r 2

4n2 +4n+2 = lim v u u t

2 4+ 4

n+ 2 n2

= √1

2. □

(12)

CHUYÊN TO ÁN - CHUYÊN BẾN TRE

Bài toán A10 (Olympic Toán Sinh viên 2011) Choα,βRthỏa mãn điều kiện

1+ 1

n α+n

<e <

1+ 1

n β+n

∀n⩾1 Tìm min|αβ|.

Đề xuất bởi Nguyễn Thái Tân, T18-21

Lấylncác vế, ta thu được

α < 1 ln

1+ 1

n

−n< β,∀n ⩾1 (*)

Ta cần khảo sát dãy số xn = 1 ln

1+ 1

n

n.

Dễ chứng minhxn là dãy số tăng.

Lại có

nlim(xn) = lim

t0

1

ln(1+t) −1 t

= lim

t0

t−ln(1+t) tln(1+t)

= lim

t0

1− 1 1+t ln(1+t) + t

t+1

=lim

t0

1 (1+t)2 1

t+1+ 1 (1+t)2

= 1

2 (theo quy tắcL’Hospital)

Tồn tại εđủ nhỏ sao cho α = x1ε, β= 1

2−εthỏa mãn (*).

Do đó

min|αβ|= 1

2−x1 = 3 2 − 1

ln 2

(13)

TỔNG HỢP CÁ C BÀI TO ÁN HA Y

Bài toán A11 (VMO 2013)

Cho dãy số thực(an)xác định bởi a1 =1và

an+1 =3− an+2 2an

với mọi n ⩾ 1. Chứng minh rằng dãy (an) có giới hạn hữu hạn. Hãy tìm giới hạn đó.

Đề xuất bởi Trần Võ Tường Vinh, 12T

Lời giải. Trước hết, bằng quy nạp, ta sẽ chứng minh

an >1, ∀n >1. (1) Thật vậy, với n = 2, ta cóu2 = 3

2 nên (*) đúng. Giả sử(1)đúng với n = k > 1, tức là ak > 1, ta cần chứng minh ak+1 =3− ak+2

2ak > 1, hay 2ak+1 > ak+2.

Xét hàm sốu(x) = 2x+1−x−2trên(1,+). Ta có

u(x) = 2x+1ln 2−1 >0, ∀x> 1.

Do đó u(x) là hàm số đồng biến, suy rau(ak) > u(1) > 0. Từ đó suy ra (*) đúng với mọin >1. Bây giờ, do an1với mọin⩾1nên

an+1 =3an+2

2an <3, ∀n⩾1.

Tiếp theo, ta sẽ chứng minh (an)là dãy tăng. Xét hàm số f(x) = 3− 2+x

2x với1< x <3. Ta có

f(x) = ln 4+xln 2−1 2x >0, nên f(x)đồng biến trên(1, 3). Ta cũng cóa2 = 3

2 > a1 nên dãy tăng. Do đó, dãy(an)đã cho tăng và bị chặn trên bởi 3 nên có giới hạn hữu hạn. Giả sử giới hạn đó làL ∈ (1, 3). Trong công thức xác định của dãy, chuyển qua giới hạn, ta có

L =3− L+2 2L .

(14)

CHUYÊN TO ÁN - CHUYÊN BẾN TRE

Ta sẽ chứng minh phương trình

x =3− x+2

2x (2)

có nghiệm duy nhất trên(1, 3). Thật vậy, xét hàm số g(x) = 3− x+2

2x −x vớix ∈ (1, 3), ta có

g(x) = ln 3+xln 2−1

2x −1 = ln 4+xln 2−12x

2x .

Ta xét tiếp hàm sốh(x) = ln 4+xln 2−1−2x, x ∈ (1, 3)thì h(x) = ln 2(12x) <0,x ∈ (1, 3), nên đây là hàm nghịch biến. Suy rah(x) <h(1) = ln 8−3 <0, hay

ln 4+xln 2−1−2x < 0

với mọix ∈ (1, 3). Do đó, hàm số g(x)nghịch biến trên(1, 3)và phương trình g(x) =0 có không quá một nghiệm. Hơn nữa, ta cóg(2) = 0nên x = 2là nghiệm duy nhất của phương trình (2). Vì vậy dãy số đã cho có giới hạn là 2. □

Bài toán A12 (Moldova 2004)

Tìm đa thức P(x)hệ số thực thỏa

(x2 +3x2+3x+2)P(x−1) = (x3−3x2+3x−2)P(x) ∀x (1)

Đề xuất bởi Nguyễn Trường Thịnh, 12T Lời giải. Ta có biến đổi sau

(1)⇔ (x+2)(x2+x+1)P(x−1) = (x−2)(x2−x+1)P(x) (2) Ta chọn

x= −2P(−2) = 0; x =0 ⇒ P(0) = 0;

x= −1⇒ P(−1) = 0; x =1 ⇒ P(1) = 0.

⇒ P(x) = (x−1)x(x+1)(x+2)Q(x), vớiQ(x) ∈ R[x]. (2) ⇒ (x+2)(x2+x+1)(x−2)(x−1)x(x+1)Q(x−1)

= (x−2)(x2−x+1)(x−1)x(x+1)(x+2)Q(x).

⇒ (x2+x+1)Q(x−1) = (x2 −x+1)Q(x) ∀x ̸= −2,−1, 0, 1, 2.

(15)

TỔNG HỢP CÁ C BÀI TO ÁN HA Y

Q(x)

x2+x+1 = Q(x−1)

(x−1)2+ (x−1) +1 ∀x ̸= −2,−1, 0, 1, 2.

Đặt R(x) = Q(x) x2+x+1.

R(x) = R(x−1) ⇒ R(x)≡ cQ(x) = c(x2 +x+1).

P(x) = c(x−1)x(x+1)(x+2)(x2+x+1), thử lại thấy thỏa mãn bài toán.

VậyP(x) = c(x−1)x(x+1)(x+2)(x2+x+1). □ Bài toán A13

ChoP(x),Q(x),R(x)là các đa thức khác hằng, có hệ số thực và thỏa mãn P(x2 −x) +xQ(x2−x) = (x2−4)R(x)với mọix ∈ .

a) Chứng minh rằng phương trình Q(x) = R(x−3) có ít nhất hai nghiệm thực phân biệt.

b) Giả sử tổng bậc của P(x),Q(x),R(x) là 5 và hệ số bậc cao nhất của R(x)là 1.

Tìm giá trị nhỏ nhất của M = P2(0) +8Q2(3).

Đề xuất bởi Đoàn Quang Đăng, 12T

Lời giải.

a) Trong đẳng thức đã cho, lần lượt thay x= 2, x =−2, ta có P(2) +2Q(2) = 0,P(6)−2Q(6) = 0.

Để có x2−x = 2, ngoài x= 2, ta còn có x = −1nên thay tiếp x =−1vào, ta được P(2)−Q(2) = −3R(−1).

Từ đó suy ra−3Q(2) = −3R(−1)hay Q(2) = R(−1).

Tương tự, thayx = 3vào, ta cóP(6) +3Q(6) = 5R(3)nênQ(6) = R(3). VậyQ(x) = R(x−3)có hai nghiệm phân biệt làx =2,x= 6.

b) Gọi m,n,p ∈ Z+ lần lượt là bậc của P(x), Q(x), R(x) thì m +n+ p = 5 và max{2m; 2n+1} = p+2. Dễ thấym =2, n =1, p =2.

ĐặtQ(x) = ax+bvới a ̸= 0.

Vì R(x−3)−Q(x) là đa thức bậc hai, có hệ số cao nhất bằng 1 và có hai nghiệm là x =2, x= 6nên R(x−3)−Q(x) = (x−2)(x−6).

Suy raR(x−3) = x2−8x+12+ax+b.

Từ đó ta tính được R(x) = (x+3)28(x+3) +12+a(x+3) +b = x2+x(a−2) + 3a+b−3.

(16)

CHUYÊN TO ÁN - CHUYÊN BẾN TRE

Trong đẳng thức đề bài cho, thay x= 0, ta có P(0) = −4R(0) = −4(3a+b−3). Suy raP2(0) +8Q2(3) = 16(3a+b−3)2+8(3a+b)2.

Đặtc =3a+b, ta cóP2(0) +8Q2(3) = 16(t−3)2+8t2 = 24(t2 −4t+6)⩾48.

Vậy giá trị nhỏ nhất của M là48, đạt được khit = 2hay3a+b = 2.

Ứng với a = 1,b = −1, ta có các đa thứcP(x) = x25x+4, Q(x) = x−1, R(x) = x2x1thỏa mãn đề bài và P2(0) +8Q2(3) = 16+8.22 =48. □

Bài toán A14 (Chọn đội tuyển VMO Hà Tĩnh 2022)

Cho đa thứcP(x)có các hệ số nguyên, giả sử các phương trình P(x) = 1,P(x) = 2 và P(x) = 3theo thứ tự mỗi phương trình có ít nhất một nghiệm nguyên lần lượt làx1,x2,x3.

a) Chứng minh rằng x1,x2,x3là các nghiệm duy nhất của các phương trình trên.

b) Chứng minh rằng phương trình P(x) = 5không có hơn một nghiệm nguyên.

Đề xuất bởi Đào Trọng Toàn, 12T

Lời giải.

a) DoP(x2) = 2⇒ P(x) = (x−x2)q(x) +2,q(x)∈ Z[x] (1) Chox1 = x2 =x3

(1 = P(x1) = (x1−x2)q(x1) +2 3 = P(x3) = (x3−x2)q(x3) +2

((x1 −x2)q(x1) = −1 (x3 −x2)q(x3) = 1 Dox1−x2, x3−x2, q(x1), q(x3) ∈ Z ⇒ x1−x2, x3−x2 ∈ {−1; 1}

Mà x1 ̸= x3 nên

"

x1−x2 =1và x3−x2 = −1 x1−x2 = −1vàx3−x2 = 1 (2) Từ đó suy ra x2 = x1+x3

2 .

Giả sử phương trình P(x) = 2 có một nghiệm nguyên x2 ̸= x2, lý luận như trên ta suy rax2 = x1+x3

2 ⇒ x2 = x2 mâu thuẫn, suy raP(x) = 2có một nghiệm nguyên duy nhất làx2.

Nếu phương trình P(x) = 1 có 1 nghiệm nguyên x1 ̸= x1, lý luận tương tự suy ra x1 = 2x2 −x3. Suy ra x1 = x1 mâu thuẫn, suy ra x1 là nghiệm nguyên duy nhất của phương trìnhP(x) = 1.

Tương tựx3 là nghiệm nguyên duy nhất của phương trìnhP(x) = 3.

b) Giả sử phương trìnhP(x) = 5có một nghiệm nguyên x5.

Trong (1) cho x = x5 suy ra 5 = P(x5) = (x5−x2)q(x5) +2 hay (x5 −x2)q(x5) = 3⇒ x5−x2 chỉ có thể lấy các giá trị±1hay±3.

(17)

TỔNG HỢP CÁ C BÀI TO ÁN HA Y

Nếux5−x2 = ±1thì theo (2)⇒

"

x5 = x1

x5 = x3 vô lý vì x5 ̸= x1 vàx5 ̸= x3. Suy rax5−x2 = ±3 (3).

Mặt khác,P(x) = (x−x3)r(x) +3, vớir(x)∈ Z[x].

Chox = x5 ⇒ (x5−x3)r(x5) = 2⇒ x5 −x3 chỉ lấy giá trị±1hay±2.

Nếux5−x3 = ±1thì mâu thuẫn với(2)(3). Suy rax5−x3 =±2 (4). Xét(2), (3), (4)ta thấy nếux1−x2 =1và x3−x2 =−1thì x5−x2 = −3.

Do đó, nghiệm nguyên x5 (nếu tồn tại) của phương trình P(x) = 5 được hoàn toàn xác định bởi các sốx1,x2,x3 duy nhất.

Suy raP(x) = 5không thể có hơn một nghiệm nguyên. □ Bài toán A15

Tìm tất cả đa thức hệ số thựcP(x)sao choP(0) = 0và

[P([P(n)])]−n+1 =2[P(n)], ∀nZ+. với[x]là phần nguyên củax.

Đề xuất bởi Đoàn Quang Đăng, 12T

Lời giải. ĐặtP(x) = akxk+Q(x),ak ̸= 0, degQ(x) <degP(x). Khi đó lim

x→+

P(x)

xk = ak ⇒ lim

n→+

[P(n)]

nk =ak. Dễ thấyk =0không thỏa.

Nếuk> 1lim

n→+

VT

nk2 = akk+1, lim

n→+

VP

nk = 2ak

( k2 =k

akk+1 =2ak không thỏa.

Nếu k = 1 ⇒ lim

n→+

VT

n = a21−1, lim

n→+

VP

n = 2a1 ⇒ a21−2a1−1 = 0 ⇒ a1 = 1±√

2.

Trường hợp 1.P(x) = 1−√ 2

x,∀x ∈ R.

Đẳng thứch

1−√ 2 h

1−√ 2

nii

−n+1= 2h

1−√ 2

ni

sai vớin =1.

Trường hợp 2.P(x) = 1+√ 2

x,∀x ∈ R.

Ta chứng minh đẳng thức[α[αn]]−n+1=2[αn]đúng vớiα21= 0,α >1.

(18)

CHUYÊN TO ÁN - CHUYÊN BẾN TRE

Thật vậy, xét

α[αn]−n+1−2[αn] = (α−2) [αn]−n+1= [αn]

α −n+1= −{αn} α +1

⇒ 0 <−1

α +1 <α[αn]−n+1−2[αn]< 1⇒ [α[αn]]−n+1 =2[αn] VậyP(x) = 1+√

2

x, ∀x ∈ Rthỏa mãn yêu cầu bài toán. □ Bài toán A16

Tìm tất cả các đa thức P(x) với hệ số thực sao cho với mọi số thực x,y,z có tổng bằng 0 thì trên mặt phẳngOxycác điểm(x,P(x)),(y,P(y)),(z,P(z))thẳng hàng.

Đề xuất bởi Đoàn Quang Đăng, 12T Lời giải.Ta sẽ chứng minhP(x) = ax3+bx+c, trong đóa,b,clà các số thực tùy ý . Rõ ràng P(x) = 1, P(x) = xthỏa mãn yêu cầu.

Các điểm(x,P(x)),(y,P(y)),(z,P(z)) thẳng hàng khi và chỉ khi (x−y)P(z) + (y−z)P(x) + (z−x)P(y) = 0.

Nếux+y+z =0thì

(x−y)z3+ (y−z)x3+ (z−x)y3 = −(x−y) (y−z) (z−x) (x+y+z) = 0 Vì thế P(x) = x3 thỏa mãn yêu cầu.

Do đóP(x) = ax3+bx+cvới a,b,ctùy ý.

Thật vậy, thayx = x, y =2x, z= −3xta được

xP(−3x) +5xP(x)−4xP(2x) = 0.

Do đó cho P(x) = a0+a1x+...+anxn, 0⩽ knta có ak

(−3)k+4.2k−5

= 0.

Với k⩾2,kchẵn, ta có(−3)k+4.2k532+4.225k5,klẻ, ta có

(−3)k+4.2k−5

= (2+1)k−4.2k+5

2k+k.2k1+ k(k−1)

2 .2k2−4.2k+5⩾

1+5

2 +5(5−1)

8 −4

.2k+5>0.

Từ đó vớik ̸=0, 1, 3 ta cóak = 0.

Vậy không còn đa thức nào khác. □

(19)

TỔNG HỢP CÁ C BÀI TO ÁN HA Y

Bài toán A17 (Bắc Ninh)

Tìm điều kiện của số dương k sao cho với mọia,b,c dương thỏa mãn abc = 1, ta luôn có

1

ak(b+c) + 1

bk(c+a) + 1

ck(a+b) ⩾ 3 2

Đề xuất bởi Nguyễn Song Thiên Long, 11T Lời giải. Xétt >0, cho a= b =t vàc = 1

t2. Bất đẳng thức trở thành 1

2t2k1+ 2t

2k

t3 +1 ⩾ 3 2 Nếuk < 1

2 thì cho t → +∞, vế trái tiến dần về 0, trong khi vế phải là hằng số và lớn hơn 0, mâu thuẫn. Do đó ta phải cók ⩾ 1

2.

Tiếp theo, ta sẽ chứng minhk ⩾ 2. Thật vậy, giả sử 1

2 ⩽ k < 2, chot → 0, vế trái tiến dần về 0, trong khi đó vế phải là hằng số lớn hơn 0, mâu thuẫn. Vậy ta phải cók⩾ 2.

Bây giờ ta sẽ chứng minh vớik⩾2thì bất đẳng thức đã cho đúng. Áp dụng bất đẳng thứcChebyshevcho hai bộ số đơn điệu cùng chiều

1

ak2, 1

bk2, 1 ck2

1 a2(b+c),

1 b2(c+a),

1 c2(a+b)

,

ta được

ak(b1+c) ⩾ 1 3

ak12

a2(b1+c)

.

Mặt khác, áp dụng bất đẳng thứcAM-GMthì 1

ak2 + 1

bk2 + 1

ck2 ⩾ √3 3

ak2bk2ck2 =3.

Kết hợp với trên, ta có 1

ak(b+c) + 1

bk(c+a) + 1

ck(a+b) ⩾ 1

a2(b+c) + 1

b2(c+a) + 1 c2(a+b). Do đó, ta chỉ cần chứng minh được

1

a2(b+c) + 1

b2(c+a) + 1

c2(a+b) ⩾ 3 2,

(20)

CHUYÊN TO ÁN - CHUYÊN BẾN TRE

hay

bc

ab+ac + ca

bc+ba + ab

ca+cb ⩾ 3 2.

Đây chính là bất đẳng thứcNesbittđược áp dụng cho ba số dươngbc,ca,ab.

Vậyk⩾ 2là tập hợp các giá trị cần tìm. □

Bài toán A18

Chon > 2và x1,x2, . . . ,xn >0thỏa mãn

n k=1

xk

! n

k

=1

1 xk

!

= n2+1.

Chứng minh rằng

n k=1

x2k

! n

k

=1

1 x2k

!

>n2 +4+ 2

n(n−1)

Đề xuất bởi Nguyễn Song Thiên Long, 11T Lời giải. Ta sẽ chứng minh bài toán bằngđồng nhất thứcvà bất đẳng thứcCauchy - Schwarz. Ta bắt đầu với biểu thức

1i

<jn

xi xj + xj

xi −2

!2

Dễ thấy

1i

<jn

xi xj + xj

xi −2

!2

=

1i<jn

x2i x2j + x

2j

xi2 −4.xi

xj −4.xj xi +6

!

=

n i=1

x2i

! n

i

=1

1 x2i

!

−4

n i=1

xi

! n

i

=1

1 xi

!

+3n2

Do

1i<jn

xi xj + xj

xi −2

!2

0nên

n i=1

x2i

! n

i

=1

1 x2i

!

n2+4.

Tuy nhiên, ta chưa có được điều phải chứng minh. Do đó, ta cần làm chặt biểu thức

1i

<jn

xi xj + xj

xi −2

!2

0

(21)

TỔNG HỢP CÁ C BÀI TO ÁN HA Y

Áp dụng bất đẳng thứcCauchy - Schwarz, ta có

1i

<jn

xi xj + xj

xi2

!2

1i<jn

xi xj + xj

xi −2

!2

2

Cn2

1i<jn

xi xj + xj

xi −2

!

= 1nên suy ra được

n i=1

x2i

! n i

=1

1 x2i

!

n2+4+ 2 n(n−1) Ta cần chứng minh đẳng thức không xảy ra.

Thật vậy, đẳng thức xảy ra khix1 = x2 = . . . = xn, điều này mâu thuẫn với giả thiết

n i=1

xi

! n i

=1

1 xi

!

= n2+1

□ Bài toán A19

Cho0 < a1a2. . .an vàana[n+1/2] ⩽1. Chứng minh rằng n

1+ n s n

i=1

ai

n i=1

1 1+ai

Đề xuất bởi thầy Mai Hữu Vinh (S11-14)

Lời giải. Với a1,a2 >0vàa1,a2 ⩽1. Ta chứng minh 2

1+√

a1a21

1+a1 + 1

1+a2 ⇔ (√

a1−√

a2)2(1−√ a1a2) (1+√

a1a2) (1+a1) (1+a2) ⩾0(đúng) Đẳng thức xảy ra khia1 = a2 hoặca1a2 = 1.

Áp dụng ta có

n i=1

1 1+ai

[n+1/2] i

=1

2 1+√

a1an+1i

Đẳng thức xảy ra khinchẵn vàa1 = a2 = . . . = an hoặca1an+1i = 1.

(22)

CHUYÊN TO ÁN - CHUYÊN BẾN TRE

Xét f(x) = 1

1+ex, ∀x⩽0,

f(x) = −ex

(1+ex)2; f

′′(x) = e

x(ex1) (1+ex)30.

Áp dụng bất đẳng thứcJensenvào hàm lõm với cácxi0vàexi = √

aian+1i ⩽1, ta có

n i=1

1 1+ai

[n+1/2] i

=1

2 1+√

aian+1in 1+ n

s n

i=1

ai .

(23)

TỔNG HỢP CÁ C BÀI TO ÁN HA Y

2 Số học

Bài toán N1 (Chọn đội tuyển VMO Hà Tĩnh 2022) Tìm tất cả các số nguyên dươngnsao cho: n3n23n+1

3n−2 là1số nguyên.

Đề xuất bởi Nguyễn Song Thiên Long, 11T

Lời giải.

Yêu cầu đề bài tương đương: “Tìm tất cả số nguyên dương nđể n3n2−1

3n−2 ∈ Z” hay

“Tìm tất cả số nguyên dươngnđể3n−2|n3n21”.

Xétn =1thấy thỏa mãn.

Xétn ⩾2. Đặtt =3n−2và gọi plà ước nguyên tố nhỏ nhất của t.

Theo đề bài ta cónt ≡1(modp)⇒ ordpn|t.

Giả sử(n,p) ̸=1, khi đó tồn tạiλ ̸=1sao cho (

λ|n λ|p ⇒

( p|n

p= λ ⇒ p| −1(vô lí) Suy ra(n,p) = 1nên theo định lý Fermat nhỏta cónp11(modp)⇒ ordpn|p1.

Nếu gọi q là ước nguyên tố của ordpn thì ta thấy q|t q < p−1 < p. Và điều này mâu thuẫn vì plà ước nguyên tố bé nhất củat.

Trường hợp này không tìm đượcnthỏa mãn đề bài.

Vậy chỉ có duy nhấtn =1thỏa mãn đề bài. □

Bài toán N2

ChoS(n)là tổng các chữ số củan. Hỏi S(2n)có thể tăng ngặt từ giá trị nào đó hay không?

Đề xuất bởi Đào Trọng Toàn, 12T Lời giải.Giả sử tồn tại mmà với mọi số tự nhiênk > m,S(k)tăng ngặt.

Ta cóS(n) ≡ n(mod9)nên ta có các bất đẳng thức sau

S(26k+1)⩾ S(26k) +1, S(26k+2) ⩾S(26k+1) +2, S(26k+3) ⩾S(26k+2) +4 S(26k+4) ⩾S(26k+3) +8, S(26k+5) ⩾S(26k+4) +7, S(26k+6) ⩾S(26k+5) +5 Từ đây dễ thấy rằngS(26k+6)⩾S(26k) +27.

Bằng quy nạp đơn giản, ta cóS(26k+6l)⩾S(26k) +27l, ∀l ∈ N.

Mặt khác, do26k+6l =82k+2l nênS(26k+6l)< 9(2k+2l).

(24)

CHUYÊN TO ÁN - CHUYÊN BẾN TRE

Ta thu được9(2k+2l) >S(26k) +27l, ∀l ∈ N.

Chol → ta thấy ngay điều này mâu thuẫn, do đó hàm S(2n) không thể tăng ngặt

từ bất kì giá trị nào. □

Bài toán N3 (Taiwan MO 1998)

Chom,nlà hai số lẻ vớim > n> 1thỏa mãn

m2−n2+1|n2−1.

Chứng minh rằngm2−n2+1là số chính phương.

Đề xuất bởi Nguyễn Song Thiên Long, 11T

Lời giải.

Bổ đề. Với n là số chính phương thì phương trình (x+y)2 = n(4xy+1) luôn có nghiệm nguyên dương.

Chứng minh. Cố định n và xét tất cả các cặp (x,y) nguyên dương thỏa mãn phương trình(x+y)2 =n(4xy+1),có nghĩa là xét tập

S= n(x,y)∈ N×N|(x+y)2 = n(4xy+1)o

Vì S là tập các cặp số nguyên dương nên luôn tồn tại một cặp (x0,y0) trong S mà x0+y0 đạt giá trị nhỏ nhất và x0y0.

Xét phương trình

(x+y0)2 = n(4xy0+1) ⇔ x2−(4n−2)y0x+y20−n =0

là một phương trình bậc hai ẩnx. Khi đó x0 là một nghiệm của phương trình trên. Như vậy theo định lýViètethì phương trình còn có một nghiệm nữa làx1, và

(x0+x1 = (4n−2)y0 x0.x1 =y20 −n

Từ hệ thức thứ nhất x0+x1 = (4n−2)y0 ta suy ra được x1 là số nguyên. Ta xét các trường hợp sau

Nếu x1 < 0thì từ hệ thức thứ hai x0.x1 = y20nta được y20n < 0 ⇒ y20 < n, ta suy ra

x21−(4n−2)y0x1+y20n = (x1+y0)2+n(−4x1y01)> 0 điều này mâu thuẫn vìx1 là nghiệm của phương trình.

Nếu x1 = 0, khi đó từ x0.x1 = y20−n ta được y20 −n = 0 ⇒ n = y20 là số chính phương.

(25)

TỔNG HỢP CÁ C BÀI TO ÁN HA Y

Nếu x1 > 0 thì ta được y20 −n > 0 ⇒ y20 > n. Khi đó (x1,y0) là một nghiệm của phương trình(x+y)2 = n(4xy+1). Theo cách chọn cặp số(x0,y0)ta có

x0+y0x1+y0 ⇒ y0x0x1

Kéo theo y20 −(4n−2)y20+y20 −n = (4−4n)y20−n ⩾ 0, điều này vô lý vì n là số nguyên dương.

Bổ đề được chứng minh hoàn tất.

Trở lại bài toán.Theo giả thiết, ta cóm2−n2+1|n2−1nên ta đượcm2 −n2+1|m2− (m2−n2+1). Từ đó suy ra(m2−n2 +1)|m2.

Từ điều trên ta suy ra tồn tạikđể

m2 = k(m2−n2+1) Chú ý rằngm2 =

m+n

2 + mn 2

2

và m2−n2 +1 = 4.m+n

2 .m−n

2 +1. Dom và n là các số nguyên dương lẻ, lại có m > n nên m+n

2 ; m−n

2 là các số nguyên dương.

Đặtx = m+n

2 ; y= m−n

2 , ta được(x+y)2 = k(4xy+1).

Như vậy ta cần chứng minhm2n2+1=4xy+1là số chính phương, tức là chỉ cần chứng minhklà số chính phương.

Áp dụng bổ đề trên, ta có ngay điều phải chứng minh. □ Bài toán N4

Cho đa thứcP(x) = xk+bk1xk1+...+b1x+1thỏa mãn i) k ⩾2.

ii) Với mọii = 1, ..,n−1ta cóbiN bi =bki.

Chứng minh rằng tồn tại vô số cặp số nguyên dương(m,n)thỏa mãn

(P(m)...n P(n)...m Đề xuất bởi Đoàn Quang Đăng, 12T Lời giải. Xét cặp (m,n) thỏa mãn yêu cầu bài toán. Không mất tổng quát giả sử m ⩾n.

Ta chứng minh cặp

m;P(m) n

cũng thỏa mãn yêu cầu bài toán.

Hiển nhiên P(m)...P(m)

n . Ta chứng minh P

P(m) n

...m.

Đặtd = (m,n). Do P(m)...nnênP(m)...d.

Mà P(m)−1= P(m)−P(0)...m nên suy rad =1.

(26)

CHUYÊN TO ÁN - CHUYÊN BẾN TRE

Ta có P

P(m) n

= P

k(m) +bk1Pk1(m).n+...+b1P(m).nk1+nk

nk .

DoP(m) ≡ 1(modm)nênP

P(m) n

.nk ≡1+bk1.n+..+b1.nk1+nk ≡ P(n) ≡0 (modm).

Suy raP

P(m) n

.nk...mmà(m,n) = 1nênP

P(m) n

...m.

Hai cặp(m,n)

m; P(m) n

khác nhau vì P(m) n > m

2

n ⩾nnênm+P(m)

n > m+n.

Do đó với cặp (m,n) thỏa mãn bài toán, ta xây dựng được cặp (M,N)thỏa mãn bài toán vàM+N > m+n.

Cuối cùng ta chỉ ra cặp(P(1); 1)thỏa mãn bài toán và theo cách trên ta thu được dãy

vô hạn cặp(m,n)thỏa mãn bài toán. □

Bài toán N5 (Putnam Exam 2008)

Cho plà số nguyên tố và f(x)là một đa thức với hệ số nguyên thỏa mãn nf(0);f(1); f(2). . . ; f(p2−1)o

là một hệ Thặng dư đầy đủ theo modulo p2. Chứng minh rằng nf(0); f(1);f(2); . . . ; f(p3−1)o là một hệ Thặng dư đầy đủ theo modulo p3.

Đề xuất bởi Đoàn Quang Thắng, Quảng Bình Lời giải. Đầu tiên ta chứng minh rằng f(x) không thể chia hết cho p với mọi x nguyên. (1)

Thật vậy nếu tồn tại một số xnguyên như thế thì hiển nhiên ta chọnysao cho y ∈ N, 0y < p,

Tài liệu tham khảo

Đề cương

Tài liệu liên quan

- Thay khối nón bởi khối trụ được tạo thành khi cắt khối cầu đã cho bởi hai mặt phẳng song song với nhau, cách đều tâm khối cầu và cùng vuông góc với đoạn

Biết rằng bác Mạnh không rút tiền ra khỏi ngân hàng thì cứ sau mỗi tháng, số tiền lãi sẽ được nhập vào vốn ban đầu (ta gọi đó là lãi kép)?. Sau một năm gửi tiền,

Thủy phân hoàn toàn este bằng dung dịch H 2 SO 4 loãng rồi cho nước brom dư vào sản phẩm thấy thu được hai chất hữu cơ chứa brom trong đó có một chất

Câu 3: Quá trình nhiều phân tử nhỏ (monome) kết hợp với nhau thành phân tử lớn (polime) đồng thời giải phóng những phân tử nước được gọi là phản ứng.. Câu

Trong các kì thi tuyển sinh lớp 10 các trường chuyên trên toàn quốc thì các bài toán về số học xuất hiện một cách đều đặn trong các đề thi với các bài toán ngày càng

Bài toán 13 (China South East Mathematical Olympiad 2021).. Chứng minh hoàn tất. a) Tìm công thức tổng quát của dãy trên... Theo nguyên lí quy nạp thì

Đối với một bài toán bất đẳng thức nhiều biến số, thông thường, ta sẽ nghĩ đến hai phương pháp là quy nạp hoặc biến đổi tương đương... Bài toán trên thực sự rất cơ bản,

Đường tròn (J ) ngoại tiếp tam giác AEF cắt (O) tại điểm thứ hai là K.. Áp dụng điều trong chứng minh định lí Brocard ta có: M P, F Q, HE đồng quy do đó: M, P, J